Đến nội dung

Hình ảnh

Cauchy-Schwarz


  • Please log in to reply
Chủ đề này có 176 trả lời

#81
banhgaongonngon

banhgaongonngon

    Thượng úy

  • Thành viên
  • 1046 Bài viết

Cho hỏi cách chứng minh hệ quả này với.


Áp dụng BĐT$Cauchy-Schwarz$ ta có

$\left [ \left ( \sqrt{b_{1}} \right )^{2}+\left ( \sqrt{b_{2}} \right )^{2}+...+\left ( \sqrt{b_{n}} \right )^{2} \right ]\left [ \left ( \frac{a_{1}}{\sqrt{b_{1}}} \right )^{2}+\left ( \frac{a_{2}}{\sqrt{b_{2}}} \right )^{2}+...+ \left ( \frac{a_{n}}{\sqrt{b_{n}}} \right )^{2}\right ]\geq (a_{1}+a_{2}+...+a_{n})^{2}$



#82
nguyencuong123

nguyencuong123

    Thiếu úy

  • Thành viên
  • 587 Bài viết
Cho 2 số thực a,b thoả mãn a > b và ab<0.
Tìm Min của A= $\left ( a-b \right )^{2}+\left ( a-b+\frac{1}{a}-\frac{1}{b} \right )^{2}$

    :icon12:  :icon12:  :icon12:   Bình minh tắt nắng trời vương vấn :icon12:  :icon12:  :icon12:       

      :icon12: Một cõi chơi vơi, ta với ta  :icon12:       

:nav: My Facebook  :nav:  

 


#83
nguyencuong123

nguyencuong123

    Thiếu úy

  • Thành viên
  • 587 Bài viết
Cho a,b,c là các số thực dương thoả mãn $a+b+c= 1$ .Chứng minh rằng :
$\frac{ab}{c+1}+\frac{bc}{a+1}+\frac{ca}{b+1}\leq \frac{1}{4}$

    :icon12:  :icon12:  :icon12:   Bình minh tắt nắng trời vương vấn :icon12:  :icon12:  :icon12:       

      :icon12: Một cõi chơi vơi, ta với ta  :icon12:       

:nav: My Facebook  :nav:  

 


#84
banhgaongonngon

banhgaongonngon

    Thượng úy

  • Thành viên
  • 1046 Bài viết

Cho a,b,c là các số thực dương thoả mãn $a+b+c= 1$ .Chứng minh rằng :
$\frac{ab}{c+1}+\frac{bc}{a+1}+\frac{ca}{b+1}\leq \frac{1}{4}$


Ta có $\sum \frac{ab}{c+1}=\sum \frac{ab}{(a+c)+(b+c)}\leq \frac{1}{4}\left ( \sum \frac{ab}{a+c}+\sum \frac{ab}{b+c} \right )=\frac{1}{4}(a+b+c)$

#85
Oral1020

Oral1020

    Thịnh To Tướng

  • Thành viên
  • 1225 Bài viết

Cho 2 số thực a,b thoả mãn a > b và ab<0.
Tìm Min của A= $\left ( a-b \right )^{2}+\left ( a-b+\frac{1}{a}-\frac{1}{b} \right )^{2}$

Đại nhé.
Theo $C-S$,ta có:
$A \ge \dfrac{(\dfrac{1}{a}-\dfrac{1}{b})^2}{2} \ge 0$
Dấu $=$ chỉ xảy ra khi $a=b$ nhưng do $a;b$ trái dấu
$\Longrightarrow a=b=0$
Mà mình thấy ngay lúc đầu thì là tổng hai bình phương luôn lơn hoặc bằng $0$,suy ra ngay cũng được mà

"If I feel unhappy,I do mathematics to become happy.


If I feel happy,I do mathematics to keep happy."

Alfréd Rényi

Hình đã gửi


#86
banhgaongonngon

banhgaongonngon

    Thượng úy

  • Thành viên
  • 1046 Bài viết

Đại nhé.
Theo $C-S$,ta có:
$A \ge \dfrac{(\dfrac{1}{a}-\dfrac{1}{b})^2}{2} \ge 0$
Dấu $=$ chỉ xảy ra khi $a=b$ nhưng do $a;b$ trái dấu
$\Longrightarrow a=b=0$
Mà mình thấy ngay lúc đầu thì là tổng hai bình phương luôn lơn hoặc bằng $0$,suy ra ngay cũng được mà


Hình như bài của e sai rồi :luoi:
$ab<0\Rightarrow a,b\neq 0$

#87
nguyencuong123

nguyencuong123

    Thiếu úy

  • Thành viên
  • 587 Bài viết

Cho các số thực không âm a,b,c thoả mãn $a^{2}+b^{2}+c^{2}=3$

Chứng minh rằng :
$\frac{a}{a^{2}+2b+3}+\frac{b}{b^{2}+2c+3}+\frac{c}{c^{2}+2a+3}\leq \frac{1}{2}$

    :icon12:  :icon12:  :icon12:   Bình minh tắt nắng trời vương vấn :icon12:  :icon12:  :icon12:       

      :icon12: Một cõi chơi vơi, ta với ta  :icon12:       

:nav: My Facebook  :nav:  

 


#88
backieuphong

backieuphong

    Binh nhất

  • Thành viên
  • 35 Bài viết
Cho 1/x + 1/y = 1/2
Tim Max
M = $\frac{1}{x^4+y^2+2xy^2}+\frac{1}{x^2+y^4+2x^2y}$

#89
Zaraki

Zaraki

    PQT

  • Phó Quản lý Toán Cao cấp
  • 4273 Bài viết

Cho x,y,z > 2 t/m $\sum \frac{1}{x}=1$
Tìm min : $P=\left ( x-2 \right )\left ( y-2 \right )\left ( z-2 \right )$

Lời giải. Đặt $x-2=a,y-2=b,z-2=c$. Khi đó thì $\frac{1}{a+2}+ \frac{1}{b+2}+ \frac{1}{c+2}=1$.
Ta cần tìm giá trị lớn nhất $P=abc$.

Hướng 1. Từ giả thiết thì $$\begin{aligned} \frac{1}{a+2} & = \left( \frac 12- \frac{1}{b+2} \right)+ \left( \frac 12 - \frac{1}{c+2} \right) \\ & = \frac{b}{2(b+2)}+ \frac{c}{2(c+2)} \\ & \ge \sqrt{ \frac{bc}{(b+2)(c+2)}} \end{aligned}$$
Tương tự thì $\frac{1}{b+2} \ge \sqrt{ \frac{ac}{(a+2)(c+2)}}, \; \frac{1}{c+2} \ge \sqrt{ \frac{ab}{(a+2)(b+2)}}$.
Nhân ba BĐT trên ta được $P=abc \le 1$.
Dấu đẳng thức xảy ra khi và chỉ khi $x=y=z=3$.

Hướng 2. Từ giả thiết ta suy ra $\frac{a}{a+2}+ \frac{b}{b+2}+ \frac{c}{c+2}=1$. Ta đặt $m= \frac{a}{a+2}, \; n= \frac{b}{b+2}, \; p= \frac{c}{c+2}$.
Khi đó $m+n+p=1$. Ta có $m=1- \frac{2}{a+2}= m+n+p- \frac{2}{a+2} \Rightarrow n+p= \frac{2}{a+2} \Rightarrow a= \frac{2}{n+p}-2= \frac{2m}{n+p}$.
Tương tự thì $b= \frac{2n}{p+m}, \; c= \frac{2p}{m+n}$. Bất đẳng thức trở thành $$\frac{2m}{n+p} \cdot \frac{2p}{m+n} \cdot \frac{2n}{m+p} \le 1 \Leftrightarrow (m+n)(n+p)(p+m) \ge 8mnp$$
Bất đẳng thức này hiển nhiên đúng theo AM-GM.
Dấu đẳng thức xảy ra khi và chỉ khi $x=y=z=3$.

Discovery is a child’s privilege. I mean the small child, the child who is not afraid to be wrong, to look silly, to not be serious, and to act differently from everyone else. He is also not afraid that the things he is interested in are in bad taste or turn out to be different from his expectations, from what they should be, or rather he is not afraid of what they actually are. He ignores the silent and flawless consensus that is part of the air we breathe – the consensus of all the people who are, or are reputed to be, reasonable.

 

Grothendieck, Récoltes et Semailles (“Crops and Seeds”). 


#90
Oral1020

Oral1020

    Thịnh To Tướng

  • Thành viên
  • 1225 Bài viết

Mọi người làm hộ mình bài này : Cho a,b,c là độ dài 3 cạch của 1 tam giác, a+b+c=2. CMR:
$\frac{52}{27}\leq a^{2}+b^{2}+c^{2}+2abc<2$

Bạn xem tại đây :P

"If I feel unhappy,I do mathematics to become happy.


If I feel happy,I do mathematics to keep happy."

Alfréd Rényi

Hình đã gửi


#91
barcavodich

barcavodich

    Sĩ quan

  • Thành viên
  • 449 Bài viết
Một BĐT khá đơn giản mời các bạn góp ý
Cho các số $a, b, c$ dương CMR
$\frac{ab^2}{a^2+2b^2+c^2}+\frac{bc^2}{b^2+2c^2+a^2}+\frac{ca^2}{c^2+2a^2+b^2}\leq \frac{a+b+c}{4}$

[topic2=''][/topic2]Music makes life more meaningful


#92
duong vi tuan

duong vi tuan

    Thượng sĩ

  • Thành viên
  • 229 Bài viết

Một BĐT khá đơn giản mời các bạn góp ý
Cho các số $a, b, c$ dương CMR
$\frac{ab^2}{a^2+2b^2+c^2}+\frac{bc^2}{b^2+2c^2+a^2}+\frac{ca^2}{c^2+2a^2+b^2}\leq \frac{a+b+c}{4}$

ta thấy : $3(ab^2+bc^2+ca^2)\leq (a+b+c)(a^2+b^2+c^2)\Leftrightarrow \sum a(a-b)^2\geq 0$ đúng .
áp dụng bdt trên và bat dang thức sv :
$\sum \frac{ab^2}{a^2+2b^2+c^2}\leq \sum \frac{9ab^2}{16(a^2+b^2+c^2)}+\sum \frac{ab^2}{16b^2}\leq \frac{3(a+b+c)(a^2+b^2+c^2)}{16(a^2+b^2+c^2)}+\frac{a+b+c}{16}=\frac{a+b+c}{4}$
NGU
Hình đã gửi

#93
Oral1020

Oral1020

    Thịnh To Tướng

  • Thành viên
  • 1225 Bài viết

Cho a,b,c > 0. Tìm min :
$P=\sqrt{\frac{a}{b+c}}+\sqrt{\frac{b}{a+c}}+\sqrt{\frac{c}{b+a}}$
Ai có đáp án cho mình xin

Bạn xem và thảo luận tại đây,theo mình biết bài này không có min.Chỉ có $>2$ thôi

"If I feel unhappy,I do mathematics to become happy.


If I feel happy,I do mathematics to keep happy."

Alfréd Rényi

Hình đã gửi


#94
Tienanh tx

Tienanh tx

    $\Omega \textbf{Bùi Tiến Anh} \Omega$

  • Thành viên
  • 360 Bài viết

Cho 2 số thực a,b thoả mãn a > b và ab<0.
Tìm Min của A= $\left ( a-b \right )^{2}+\left ( a-b+\frac{1}{a}-\frac{1}{b} \right )^{2}$

$\oplus$ Ta có: $\left ( a-b \right )^{2}+\left ( a-b+\frac{1}{a}-\frac{1}{b} \right )^{2} = \dfrac{(a+b)^2(2a^2b^2-2ab+1)}{a^2b^2} \ge 0$ (Do $ab<0$ $\Longrightarrow$ $-2ab \ge 0$)

Dấu $"="$ $\Longleftrightarrow$ $a=b$

$\cdot$ $( - 1) = {( - 1)^5} = {( - 1)^{2.\frac{5}{2}}} = {\left[ {{{( - 1)}^2}} \right]^{\frac{5}{2}}} = {1^{\frac{5}{2}}} =\sqrt{1}= 1$

$\cdot$ $\dfrac{0}{0}=\dfrac{100-100}{100-100}=\dfrac{10.10-10.10}{10.10-10.10}=\dfrac{10^2-10^2}{10(10-10)}=\dfrac{(10-10)(10+10)}{10(10-10)}=\dfrac{20}{10}=2$

$\cdot$ $\pi\approx 2^{5^{0,4}}-0,6-\left(\frac{0,3^{9}}{7}\right)^{0,8^{0,1}}$

$\cdot$ $ - 2 = \sqrt[3]{{ - 8}} = {( - 8)^{\frac{1}{3}}} = {( - 8)^{\frac{2}{6}}} = {\left[ {{{( - 8)}^2}} \right]^{\frac{1}{6}}} = {64^{\frac{1}{6}}} = \sqrt[6]{{64}} = 2$

 

 

 

 


#95
nguyensidang

nguyensidang

    Binh nhất

  • Thành viên
  • 32 Bài viết
các bác giúp em với em đăng bài này lên mà ko ai trả lời:
1)Cho a,b,c là các số thưc dương thoả mãn $a^{3}+b^{3}+c^{3}=3$
Tìm min của biểu thức sau:

F=$\sum \frac{bc}{a\sqrt{a}}$

2)Cho 3 số thực dương a,b,c.Thoả mãn $a^{3}+b^{3}+c^{3}=3$
Chứng minh rằng:$\sum ab(c+2)(c-1)$$\leq 0$

Bài viết đã được chỉnh sửa nội dung bởi nguyensidang: 13-03-2013 - 11:23


#96
ledinhmanh2202

ledinhmanh2202

    Lính mới

  • Thành viên
  • 6 Bài viết
2) Bất đẩng thức tương đương với abc(a+b+c+3)$\leq$2(ab+bc+ca) (1)
Áp dụng BĐT Holder ta có (a+b+c)$ ^{3}$$\leq$9(a$^{3}$+b$^{3}$+c$^{3}$) suy ra a+b+c$\leq 3$
theo BĐT AM-GM ta có 3abc$\leq (a^{3}+b^{3}+c^{3})$ $\Rightarrow abc\leq $1$\Rightarrow$$6abc\leq 6abc^{\frac{2}{3}}\leq 2(ab+bc+ca)$
mà VT của (1)$\leq $6abc
Suy ra ĐPCM

Bài viết đã được chỉnh sửa nội dung bởi ledinhmanh2202: 13-03-2013 - 11:41


#97
ledinhmanh2202

ledinhmanh2202

    Lính mới

  • Thành viên
  • 6 Bài viết
Tìm GTLN của A=$\sum \sqrt{\frac{2a}{b+c}}$
(với a,b,c>0)

Bài viết đã được chỉnh sửa nội dung bởi ledinhmanh2202: 15-03-2013 - 11:47


#98
Oral1020

Oral1020

    Thịnh To Tướng

  • Thành viên
  • 1225 Bài viết
Dự đoán là $max$ A sẽ là $3$ khi $a=b=c$
Áp dụng bất đẳng thức $AM-GM$,ta có:
$\sqrt{1.\dfrac{2a}{b+c}} \le \dfrac{2}{1+\dfrac{b+c}{2a}}=\dfrac{4a}{2a+b+c}$
Tương tự và cộng lại,ta cần chứng minh:
$\sum \dfrac{a}{2a+b+c}$
Chuẩn hóa $a+b+c=1$,ta có:
$\sum \dfrac{a}{1+a} \le \dfrac{3}{4}$
$\Longleftrightarrow 5(ab+bc+ca)+9abc \le 2$ (Dễ thấy đúng)

"If I feel unhappy,I do mathematics to become happy.


If I feel happy,I do mathematics to keep happy."

Alfréd Rényi

Hình đã gửi


#99
nguyensidang

nguyensidang

    Binh nhất

  • Thành viên
  • 32 Bài viết

Tìm GTLN của A=$\sum \sqrt{\frac{2a}{b+c}}$
(với a,b,c>0)

em có cách khác nè:
$\sum \sqrt{\frac{2a}{b+c}}=\sum \sqrt{\frac{2a(c+a)}{(b+c)(c+a)}}\leq \sqrt{(\sum c+a)(\sum \frac{2a}{(c+a)(b+c)})}$$=\sqrt{8(a+b+c)\frac{ab+bc+ca}{(a+b)(b+c)(c+a)}}$
Ta có:
$\Leftrightarrow \sum a(b-c)^{2}$
$\Leftrightarrow \sum$sym$a^{2}b$-6abc$\geq 0$
$\Leftrightarrow 24abc+8\sum$sym$a^{2}b$$\leq$$18abc+9\sum$sym$a^{2}b$
8(a+b+c)(ab+bc+ca)$\leq$9(a+b)(b+c)(c+a)
$\Leftrightarrow \sqrt{8(a+b+c)\frac{ab+bc+ca}{(a+b)(b+c)(c+a)}}\leq 3$
Vậy Min A=3 xảy ra $\Leftrightarrow a=b=c$

Bài viết đã được chỉnh sửa nội dung bởi nguyensidang: 16-03-2013 - 17:18


#100
nhatquangsin

nhatquangsin

    Thượng sĩ

  • Thành viên
  • 238 Bài viết

CM:$\sqrt{x+y}+\sqrt{z+t}\leq \sqrt{xz+yt}$

Thấy cái BĐT quen quen, đang làm bài thì tìm ra nhưng không biết cm






1 người đang xem chủ đề

0 thành viên, 1 khách, 0 thành viên ẩn danh